General derivative of the exponential operator w.r.t. a parameterDerivative of the product of operators and...

Does a star need to be inside a galaxy?

A dragon's soul trapped in a ring of mind shielding wants a new body; what magic could enable her to do so?

Workplace intimidation due to child's chronic health condition

Ethernet cable only works in certain positions

boss asked me to sign a resignation paper without a date on it along with my new contract

Why is Shelob considered evil?

Uncountable set with a non-discrete metric

How to play song that contains one guitar when we have two guitarists (or more)?

Apparently I’m calling random numbers but there's nothing in the call log

Why did Tywin never remarry?

Does the phrase がんばする makes sense?

Why do most space probes survive for far longer than they were designed for?

Is candidate anonymity at all practical?

How to not forget my phone in the bathroom?

Does human life have innate value over that of other animals?

Why is Bernie Sanders maximum accepted donation on actblue $5600?

Does a code with length 6, size 32 and distance 2 exist?

How can I get results over the whole domain of my non-linear differential equations?

How to know if I am a 'Real Developer'

How can guns be countered by melee combat without raw-ability or exceptional explanations?

Is the "hot network questions" element on Stack Overflow a dark pattern?

Can I legally make a website about boycotting a certain company?

Symbolism of number of crows?

Why didn't Lorentz conclude that no object can go faster than light?



General derivative of the exponential operator w.r.t. a parameter


Derivative of the product of operators and Derivative of exponentialDerivative of the product of operators and Derivative of exponentialRelationship between a formal vector derivative and time evolution of an operatorWhat is the commutator of the exponential derivative operator and the exponential position operator?Problem understanding electromagnetic interaction with matter (non-relativistic QED)How to obtain a two mode Squeezed state from two mode squeezed vacuum?Why the Kerr non-linearity is $a^dagger a^dagger a a$Covariant derivative vs partial derivative (in the context of Killing vectors)Physical explanation of the characteristics of the order parameter of the transverse Ising ModelTrace over configuration basisProof $exp(-beta H)$ trace-class operator













3












$begingroup$


I am interested in the calculation of the general $N$th derivative w.r.t. a parameter $lambda$ of a quantum mechanical exponential operator with the following structure:
begin{equation*}
frac{mathrm d^N}{mathrm d lambda^N} e^{-beta hat H(lambda)}
end{equation*}

In the case $n=1$ this reads as
begin{equation*}
sum_{n=0}^{+infty}sum_{m=0}^{n-1}frac{1}{n!}[hat H(lambda)]^{m}frac{mathrm dhat H(lambda)}{mathrm d lambda}[hat H(lambda)]^{n-m-1}
end{equation*}

Is there a compact fashion to rearrange the expression above for the general $N$th derivative?










share|cite|improve this question











$endgroup$

















    3












    $begingroup$


    I am interested in the calculation of the general $N$th derivative w.r.t. a parameter $lambda$ of a quantum mechanical exponential operator with the following structure:
    begin{equation*}
    frac{mathrm d^N}{mathrm d lambda^N} e^{-beta hat H(lambda)}
    end{equation*}

    In the case $n=1$ this reads as
    begin{equation*}
    sum_{n=0}^{+infty}sum_{m=0}^{n-1}frac{1}{n!}[hat H(lambda)]^{m}frac{mathrm dhat H(lambda)}{mathrm d lambda}[hat H(lambda)]^{n-m-1}
    end{equation*}

    Is there a compact fashion to rearrange the expression above for the general $N$th derivative?










    share|cite|improve this question











    $endgroup$















      3












      3








      3


      0



      $begingroup$


      I am interested in the calculation of the general $N$th derivative w.r.t. a parameter $lambda$ of a quantum mechanical exponential operator with the following structure:
      begin{equation*}
      frac{mathrm d^N}{mathrm d lambda^N} e^{-beta hat H(lambda)}
      end{equation*}

      In the case $n=1$ this reads as
      begin{equation*}
      sum_{n=0}^{+infty}sum_{m=0}^{n-1}frac{1}{n!}[hat H(lambda)]^{m}frac{mathrm dhat H(lambda)}{mathrm d lambda}[hat H(lambda)]^{n-m-1}
      end{equation*}

      Is there a compact fashion to rearrange the expression above for the general $N$th derivative?










      share|cite|improve this question











      $endgroup$




      I am interested in the calculation of the general $N$th derivative w.r.t. a parameter $lambda$ of a quantum mechanical exponential operator with the following structure:
      begin{equation*}
      frac{mathrm d^N}{mathrm d lambda^N} e^{-beta hat H(lambda)}
      end{equation*}

      In the case $n=1$ this reads as
      begin{equation*}
      sum_{n=0}^{+infty}sum_{m=0}^{n-1}frac{1}{n!}[hat H(lambda)]^{m}frac{mathrm dhat H(lambda)}{mathrm d lambda}[hat H(lambda)]^{n-m-1}
      end{equation*}

      Is there a compact fashion to rearrange the expression above for the general $N$th derivative?







      quantum-mechanics operators mathematical-physics hamiltonian differentiation






      share|cite|improve this question















      share|cite|improve this question













      share|cite|improve this question




      share|cite|improve this question








      edited 34 mins ago









      Qmechanic

      105k121881202




      105k121881202










      asked 1 hour ago









      GrazGraz

      363




      363






















          1 Answer
          1






          active

          oldest

          votes


















          4












          $begingroup$

          Hint: Use repeatedly the identity for the 1st derivative
          $$ frac{d}{dlambda}e^{that{A}} ~=~ int_0^t!dt_1~ e^{(t-t_1)hat{A}}frac{dhat{A}}{dlambda}e^{t_1hat{A}}~=~ iint_{mathbb{R}^2_+}!dt_1~dt_2~delta(t!-!t_1!-!t_2)~ e^{t_2hat{A}}frac{dhat{A}}{dlambda}e^{t_1hat{A}} , qquad t~in~mathbb{R}_+ .tag{1} $$
          For a proof of eq. (1) see e.g. my Phys.SE answer here. Then the 2nd derivative becomes
          $$begin{align} frac{d^2}{dlambda^2}e^{that{A}}
          &~=~2iiint_{mathbb{R}^3_+}!dt_1~dt_2~dt_3~delta(t!-!t_1!-!t_2!-!t_3)~ e^{t_3hat{A}}frac{dhat{A}}{dlambda}e^{t_2hat{A}}frac{dhat{A}}{dlambda}e^{t_1hat{A}} cr
          &~+~ iint_{mathbb{R}^2_+}!dt_1~dt_2~delta(t!-!t_1!-!t_2)~ e^{t_2hat{A}}frac{d^2hat{A}}{dlambda^2}e^{t_1hat{A}} , qquad t~in~mathbb{R}_+ ,end{align}tag{2} $$

          and so forth.






          share|cite|improve this answer











          $endgroup$













            Your Answer





            StackExchange.ifUsing("editor", function () {
            return StackExchange.using("mathjaxEditing", function () {
            StackExchange.MarkdownEditor.creationCallbacks.add(function (editor, postfix) {
            StackExchange.mathjaxEditing.prepareWmdForMathJax(editor, postfix, [["$", "$"], ["\\(","\\)"]]);
            });
            });
            }, "mathjax-editing");

            StackExchange.ready(function() {
            var channelOptions = {
            tags: "".split(" "),
            id: "151"
            };
            initTagRenderer("".split(" "), "".split(" "), channelOptions);

            StackExchange.using("externalEditor", function() {
            // Have to fire editor after snippets, if snippets enabled
            if (StackExchange.settings.snippets.snippetsEnabled) {
            StackExchange.using("snippets", function() {
            createEditor();
            });
            }
            else {
            createEditor();
            }
            });

            function createEditor() {
            StackExchange.prepareEditor({
            heartbeatType: 'answer',
            autoActivateHeartbeat: false,
            convertImagesToLinks: false,
            noModals: true,
            showLowRepImageUploadWarning: true,
            reputationToPostImages: null,
            bindNavPrevention: true,
            postfix: "",
            imageUploader: {
            brandingHtml: "Powered by u003ca class="icon-imgur-white" href="https://imgur.com/"u003eu003c/au003e",
            contentPolicyHtml: "User contributions licensed under u003ca href="https://creativecommons.org/licenses/by-sa/3.0/"u003ecc by-sa 3.0 with attribution requiredu003c/au003e u003ca href="https://stackoverflow.com/legal/content-policy"u003e(content policy)u003c/au003e",
            allowUrls: true
            },
            noCode: true, onDemand: true,
            discardSelector: ".discard-answer"
            ,immediatelyShowMarkdownHelp:true
            });


            }
            });














            draft saved

            draft discarded


















            StackExchange.ready(
            function () {
            StackExchange.openid.initPostLogin('.new-post-login', 'https%3a%2f%2fphysics.stackexchange.com%2fquestions%2f461860%2fgeneral-derivative-of-the-exponential-operator-w-r-t-a-parameter%23new-answer', 'question_page');
            }
            );

            Post as a guest















            Required, but never shown

























            1 Answer
            1






            active

            oldest

            votes








            1 Answer
            1






            active

            oldest

            votes









            active

            oldest

            votes






            active

            oldest

            votes









            4












            $begingroup$

            Hint: Use repeatedly the identity for the 1st derivative
            $$ frac{d}{dlambda}e^{that{A}} ~=~ int_0^t!dt_1~ e^{(t-t_1)hat{A}}frac{dhat{A}}{dlambda}e^{t_1hat{A}}~=~ iint_{mathbb{R}^2_+}!dt_1~dt_2~delta(t!-!t_1!-!t_2)~ e^{t_2hat{A}}frac{dhat{A}}{dlambda}e^{t_1hat{A}} , qquad t~in~mathbb{R}_+ .tag{1} $$
            For a proof of eq. (1) see e.g. my Phys.SE answer here. Then the 2nd derivative becomes
            $$begin{align} frac{d^2}{dlambda^2}e^{that{A}}
            &~=~2iiint_{mathbb{R}^3_+}!dt_1~dt_2~dt_3~delta(t!-!t_1!-!t_2!-!t_3)~ e^{t_3hat{A}}frac{dhat{A}}{dlambda}e^{t_2hat{A}}frac{dhat{A}}{dlambda}e^{t_1hat{A}} cr
            &~+~ iint_{mathbb{R}^2_+}!dt_1~dt_2~delta(t!-!t_1!-!t_2)~ e^{t_2hat{A}}frac{d^2hat{A}}{dlambda^2}e^{t_1hat{A}} , qquad t~in~mathbb{R}_+ ,end{align}tag{2} $$

            and so forth.






            share|cite|improve this answer











            $endgroup$


















              4












              $begingroup$

              Hint: Use repeatedly the identity for the 1st derivative
              $$ frac{d}{dlambda}e^{that{A}} ~=~ int_0^t!dt_1~ e^{(t-t_1)hat{A}}frac{dhat{A}}{dlambda}e^{t_1hat{A}}~=~ iint_{mathbb{R}^2_+}!dt_1~dt_2~delta(t!-!t_1!-!t_2)~ e^{t_2hat{A}}frac{dhat{A}}{dlambda}e^{t_1hat{A}} , qquad t~in~mathbb{R}_+ .tag{1} $$
              For a proof of eq. (1) see e.g. my Phys.SE answer here. Then the 2nd derivative becomes
              $$begin{align} frac{d^2}{dlambda^2}e^{that{A}}
              &~=~2iiint_{mathbb{R}^3_+}!dt_1~dt_2~dt_3~delta(t!-!t_1!-!t_2!-!t_3)~ e^{t_3hat{A}}frac{dhat{A}}{dlambda}e^{t_2hat{A}}frac{dhat{A}}{dlambda}e^{t_1hat{A}} cr
              &~+~ iint_{mathbb{R}^2_+}!dt_1~dt_2~delta(t!-!t_1!-!t_2)~ e^{t_2hat{A}}frac{d^2hat{A}}{dlambda^2}e^{t_1hat{A}} , qquad t~in~mathbb{R}_+ ,end{align}tag{2} $$

              and so forth.






              share|cite|improve this answer











              $endgroup$
















                4












                4








                4





                $begingroup$

                Hint: Use repeatedly the identity for the 1st derivative
                $$ frac{d}{dlambda}e^{that{A}} ~=~ int_0^t!dt_1~ e^{(t-t_1)hat{A}}frac{dhat{A}}{dlambda}e^{t_1hat{A}}~=~ iint_{mathbb{R}^2_+}!dt_1~dt_2~delta(t!-!t_1!-!t_2)~ e^{t_2hat{A}}frac{dhat{A}}{dlambda}e^{t_1hat{A}} , qquad t~in~mathbb{R}_+ .tag{1} $$
                For a proof of eq. (1) see e.g. my Phys.SE answer here. Then the 2nd derivative becomes
                $$begin{align} frac{d^2}{dlambda^2}e^{that{A}}
                &~=~2iiint_{mathbb{R}^3_+}!dt_1~dt_2~dt_3~delta(t!-!t_1!-!t_2!-!t_3)~ e^{t_3hat{A}}frac{dhat{A}}{dlambda}e^{t_2hat{A}}frac{dhat{A}}{dlambda}e^{t_1hat{A}} cr
                &~+~ iint_{mathbb{R}^2_+}!dt_1~dt_2~delta(t!-!t_1!-!t_2)~ e^{t_2hat{A}}frac{d^2hat{A}}{dlambda^2}e^{t_1hat{A}} , qquad t~in~mathbb{R}_+ ,end{align}tag{2} $$

                and so forth.






                share|cite|improve this answer











                $endgroup$



                Hint: Use repeatedly the identity for the 1st derivative
                $$ frac{d}{dlambda}e^{that{A}} ~=~ int_0^t!dt_1~ e^{(t-t_1)hat{A}}frac{dhat{A}}{dlambda}e^{t_1hat{A}}~=~ iint_{mathbb{R}^2_+}!dt_1~dt_2~delta(t!-!t_1!-!t_2)~ e^{t_2hat{A}}frac{dhat{A}}{dlambda}e^{t_1hat{A}} , qquad t~in~mathbb{R}_+ .tag{1} $$
                For a proof of eq. (1) see e.g. my Phys.SE answer here. Then the 2nd derivative becomes
                $$begin{align} frac{d^2}{dlambda^2}e^{that{A}}
                &~=~2iiint_{mathbb{R}^3_+}!dt_1~dt_2~dt_3~delta(t!-!t_1!-!t_2!-!t_3)~ e^{t_3hat{A}}frac{dhat{A}}{dlambda}e^{t_2hat{A}}frac{dhat{A}}{dlambda}e^{t_1hat{A}} cr
                &~+~ iint_{mathbb{R}^2_+}!dt_1~dt_2~delta(t!-!t_1!-!t_2)~ e^{t_2hat{A}}frac{d^2hat{A}}{dlambda^2}e^{t_1hat{A}} , qquad t~in~mathbb{R}_+ ,end{align}tag{2} $$

                and so forth.







                share|cite|improve this answer














                share|cite|improve this answer



                share|cite|improve this answer








                edited 26 mins ago

























                answered 35 mins ago









                QmechanicQmechanic

                105k121881202




                105k121881202






























                    draft saved

                    draft discarded




















































                    Thanks for contributing an answer to Physics Stack Exchange!


                    • Please be sure to answer the question. Provide details and share your research!

                    But avoid



                    • Asking for help, clarification, or responding to other answers.

                    • Making statements based on opinion; back them up with references or personal experience.


                    Use MathJax to format equations. MathJax reference.


                    To learn more, see our tips on writing great answers.




                    draft saved


                    draft discarded














                    StackExchange.ready(
                    function () {
                    StackExchange.openid.initPostLogin('.new-post-login', 'https%3a%2f%2fphysics.stackexchange.com%2fquestions%2f461860%2fgeneral-derivative-of-the-exponential-operator-w-r-t-a-parameter%23new-answer', 'question_page');
                    }
                    );

                    Post as a guest















                    Required, but never shown





















































                    Required, but never shown














                    Required, but never shown












                    Required, but never shown







                    Required, but never shown

































                    Required, but never shown














                    Required, but never shown












                    Required, but never shown







                    Required, but never shown







                    Popular posts from this blog

                    ORA-01691 (unable to extend lob segment) even though my tablespace has AUTOEXTEND onORA-01692: unable to...

                    Always On Availability groups resolving state after failover - Remote harden of transaction...

                    Circunscripción electoral de Guipúzcoa Referencias Menú de navegaciónLas claves del sistema electoral en...